Informatik/Stochastik - Übungsklausur.tex
2022-05-16 21:07:23 +02:00

400 lines
19 KiB
TeX

\documentclass[10pt, a4paper]{exam}
\printanswers % Comment this line to hide the answers
\usepackage[utf8]{inputenc}
\usepackage[T1]{fontenc}
\usepackage[ngerman]{babel}
\usepackage{listings}
\usepackage{float}
\usepackage{graphicx}
\usepackage{color}
\usepackage{listings}
\usepackage[dvipsnames]{xcolor}
\usepackage{tabularx}
\usepackage{geometry}
\usepackage{color,graphicx,overpic}
\usepackage{amsmath,amsthm,amsfonts,amssymb}
\usepackage{tabularx}
\usepackage{listings}
\usepackage[many]{tcolorbox}
\usepackage{multicol}
\usepackage{hyperref}
\usepackage{pgfplots}
\usepackage{bussproofs}
\pgfplotsset{compat=1.8}
\usepgfplotslibrary{statistics}
\renewcommand{\solutiontitle}{\noindent\textbf{Antwort}: }
\SolutionEmphasis{\small}
\geometry{top=1cm,left=1cm,right=1cm,bottom=1cm}
\pdfinfo{
/Title (Stochastik - Übungsklausur)
/Creator (TeX)
/Producer (pdfTeX 1.40.0)
/Author (Robert Jeutter)
/Subject ()
}
\title{Stochastik - Übungsklausur}
\author{}
\date{}
% Turn off header and footer
\pagestyle{empty}
% Don't print section numbers
\setcounter{secnumdepth}{0}
% Don't print section numbers
\setcounter{secnumdepth}{0}
\newtcolorbox{myboxii}[1][]{
breakable,
freelance,
title=#1,
colback=white,
colbacktitle=white,
coltitle=black,
fonttitle=\bfseries,
bottomrule=0pt,
boxrule=0pt,
colframe=white,
overlay unbroken and first={
\draw[red!75!black,line width=3pt]
([xshift=5pt]frame.north west) --
(frame.north west) --
(frame.south west);
\draw[red!75!black,line width=3pt]
([xshift=-5pt]frame.north east) --
(frame.north east) --
(frame.south east);
},
overlay unbroken app={
\draw[red!75!black,line width=3pt,line cap=rect]
(frame.south west) --
([xshift=5pt]frame.south west);
\draw[red!75!black,line width=3pt,line cap=rect]
(frame.south east) --
([xshift=-5pt]frame.south east);
},
overlay middle and last={
\draw[red!75!black,line width=3pt]
(frame.north west) --
(frame.south west);
\draw[red!75!black,line width=3pt]
(frame.north east) --
(frame.south east);
},
overlay last app={
\draw[red!75!black,line width=3pt,line cap=rect]
(frame.south west) --
([xshift=5pt]frame.south west);
\draw[red!75!black,line width=3pt,line cap=rect]
(frame.south east) --
([xshift=-5pt]frame.south east);
},
}
\begin{document}
\begin{myboxii}[Disclaimer]
Aufgaben aus dieser Vorlage stammen aus der Vorlesung \textit{Stochastik} und wurden zu Übungszwecken verändert oder anders formuliert! Für die Korrektheit der Lösungen wird keine Gewähr gegeben. Für die Klausur sind keinerlei Hilfsmittel wie Skript, Bücher oder Taschenrechner zulässig.
\end{myboxii}
\begin{questions}
\question Aufgabe 1: Laplace-Verteilung\\
Sie haben zwei Tetraeder zur Verfügung, deren Flächen jeweils mit den Augenzahlen 1, 2, 3 und 4 beschriftet sind. Beide Tetraeder werden geworfen und aus den beiden geworfenen Augenzahlen wird der Absolutbetrag ihrer Differenz, diesen nennen wir $D$, ermittelt.$D$ kann also die Werte 0, 1, 2 und 3 annehmen.
\begin{parts}
\part Geben Sie einen Wahrscheinlichkeitsraum für ein geeignetes Laplace-Experiment an und definieren Sie auf diesem Wahrscheinlichkeitsraum $D$ durch Angabe der Abbildungsvorschrift als Zufallsvariable. Vergessen Sie nicht zu begründen, warum es sich bei Ihrem gewählten Experiment um ein Laplace-Experiment handelt. \textbf{(5 Punkte)}\\
\begin{solution}
Es handelt sich um ein Laplace Experiment, da jedes Elementarereignis die gleiche Wahrscheinlichkeit $P(E)=\frac{|E|}{|D|}$ hat, mit Wahrscheinlichkeitsraum $(\Omega,\sum,P)$, $\sum=P(\Omega0)$, $\Omega= \{1,2,3,4\}^2$ und $D(\{\omega_1,\omega_2\})=|\omega_1-\omega_2|$
\end{solution}
\part Berechnen Sie die Verteilung von $D$. Ist $D$ Laplaceverteilt? Begründen Sie Ihre Antwort. \textbf{(3 Punkte)}\\
\begin{solution}
$$D(\omega)=\begin{cases}
0 \quad\text{ für } \{1,1\},\{2,2\},\{3,3\}, \{4,4\} \quad P_D({0})=\frac{4}{16} \\
1 \quad\text{ für } \{1,2\},\{2,3\},\{3,4\}, \{4,3\}, \{3,2\}, \{2,1\} \quad P_D({1})=\frac{6}{16} \\
2 \quad\text{ für } \{1,3\},\{2,4\},\{4,2\},\{3,1\} \quad P_D({2})=\frac{4}{16} \\
3 \quad\text{ für } \{1,4\},\{4,1\} \quad P_D({3})=\frac{2}{16}
\end{cases}$$ \\
Durch die unterschiedliche Wahrscheinlichkeitsverteilung der Ergebnisse ist $D$ nicht Laplaceverteilt.
\end{solution}
\part Berechnen Sie den Erwartungswert von $D$. \textbf{(2 Punkte)}\\
\begin{solution}
$$\mu_D =E(X)=\sum_i D_i*P(X=D_i) = 0*\frac{4}{16} + 1*\frac{6}{16} + 2*\frac{4}{16} + 3*\frac{2}{16} = 1,25$$
\end{solution}
\end{parts}
\question Aufgabe 2: Binomial-Verteilung
\begin{parts}
\part Geben Sie mit Hilfe Bernoulliverteilter Zufallsvariablen $Z_1,Z_2,...$ eine Zufallsvariable $X$ an, welche Binom $(n,p)$-verteilt ist. Welche Voraussetzungen müssen $Z_1,Z_2,...$ erfüllen? Was modelliert Binom $(n,p)$ anschaulich? \textbf{(3 Punkte)}\\
\begin{solution}
Eine Binominalverteilung mit Parametern n,p gilt bei $P(n,p,k)=\binom{n}{k}*p^k*(1-p)^{n-k}$ mit $k=Z_1,Z_2,...)$. Die Zufallsvariablen $Z_1,Z_2,...$ müssen dafür Ganzzahlig und Positiv sein. \\
Ein anschauliches Binom ist das ($n$-) mehrmalige Werfen einer Münze, mit Ergebnis Erfolg ($p=0,5$) (Wappen) oder Misserfolg (Zahl).
\end{solution}
\part Bestimmen Sie basierend auf $X$ den Maximum-Likelihood-Schätzer $\hat{p}$ für $p$. Existiert dieser stets eindeutig? \textbf{(5 Punkte)}\\
\begin{solution}
$L:\Theta\rightarrow [0;1],\upsilon\rightarrow p(x|\upsilon)$ wobei $\Theta$ der Parameterraum ist.
$\hat{p}= L(X)=\frac{1}{2}^k *(-\frac{1}{2})^{n-k}$
Da der Schätzer eine Wahrscheinlichkeits-parabel abzeichnet, ist immer ein Wert als Maximum möglich.
\end{solution}
\part Berechnen Sie den MSE (mean squared error) von $\hat{p}$. Ist $\hat{p}$ unverzerrt? \textbf{(2 Punkte)}\\
\begin{solution}
$MSE(T,\upsilon)=E_{\upsilon}((T-g(\upsilon))^2) = Var_{\upsilon}(T)+(B_T(\upsilon))^2$
Da die Zufallsvariablen bernoulliverteilt sind, ist die Verzerrung $B_T=0$ und die Varianz 1: $MSE(\hat{p})=\frac{1}{n}$
\end{solution}
\end{parts}
\question Aufgabe 3: Geometrische Verteilung
\begin{parts}
\part Geben Sie die Wahrscheinlichkeitsfunktion zur geometrischen Verteilung mit Parameter $p$ an. Welche Werte darf $p$ annehmen? \textbf{(2 Punkte)}\\
\begin{solution}
$$m_X(s)=\frac{pe^s}{1-(1-p)e^s}$$ mit $p=[0,\infty]$
\end{solution}
\part Was modelliert die geometrische Verteilung mit Parameter $p$? \textbf{(1 Punkt)}\\
\begin{solution}
die Wahrscheinlichkeitsverteilung der Anzahl X der Bernoulli-Versuche, die notwendig sind, um einen Erfolg zu haben. Diese Verteilung ist auf der Menge $\mathbb{N}$ definiert.
\end{solution}
\part Sie beobachten $X_1,X_2,...,X_n$ unabhängig und identisch verteilte Zufallsgrößen, die jeweils eine Geom(p)-Verteilung besitzen. Bestimmen Sie den Momentenschätzer $\hat{p}$ für $p$. \textbf{(3 Punkte)}\\
\begin{solution}
geometrische Verteilung: $f(x)=(1-p)^{x-1}*p$
$m_1=\hat{m}_1: \hat{\mu}=\frac{1}{n} \sum_{i=1}^{n} x_i$ und $m_2=\hat{m_2}:\hat{\sigma}^2+\hat{\mu}^2 =\frac{1}{n} \sum_{i=1}^{n}$
$\hat{p} = \hat{\sigma}^2=\frac{1}{n}\sum_{i=1}^n (x_i-\bar{x})^2$
\end{solution}
\part Ist $\hat{p}$ unverzerrt? \textbf{(2 Punkte)}\\
\begin{solution}
Eine Verteilung wird als verzerrt bezeichnet, wenn sich die Datenpunkte mehr zu einer Seite der Skala als zur anderen gruppieren und eine Kurve erzeugen, die nicht symmetrisch ist. Mit anderen Worten, die rechte und die linke Seite der Verteilung sind unterschiedlich geformt. Die vorliegende Verteilung ist unverzerrt.
\end{solution}
\part Ist $\hat{p}$ konsistent? \textbf{(2 Punkte)}\\
\begin{solution}
In der Statistik ist ein konsistenter Schätzer eine Regel zum Berechnen von Schätzungen eines Parameters $p$ mit der Eigenschaft, dass die resultierende Folge von Schätzungen mit zunehmender Wahrscheinlichkeit der Anzahl der verwendeten Datenpunkte in der Wahrscheinlichkeit gegen $p$ konvergiert.
Dies ist nicht der Fall.
\end{solution}
\end{parts}
\question Aufgabe 4: Exponentialverteilung
Gegeben sei eine Zufallsvariable $X \sim Exp(2)$.
\begin{parts}
\part Bestimmen Sie den Median und geben Sie den Erwartungswert von $X$ an. Vergleichen Sie die beiden Werte und erklären Sie einen eventuell vorhandenen Unterschied zwischen den beiden Ergebnissen. \textbf{(3 Punkte)}\\
\begin{solution}
Median $x_med=\frac{ln 2}{\lambda}=\frac{ln 2}{2} = 0,346574$
Erwartungswert $E(x)=\frac{1}{\lambda}= \frac{1}{2} ) = 0,5$
\end{solution}
Für die weitere Rechnung dürfen Sie ohne Nachweis benutzen, dass eine Zufallsvariable $V\sim Exp(p)$ die Varianz $Var(V) =\frac{1}{p^2}$ besitzt. Seien nun $X_{i,i}\in N$ unabhängig und identisch $Exp(2)$-verteilte Zufallsgrößen und $\bar{X}_n \frac{1}{n}\sum_{i=1}^{n} X_{i,n}\in N$ die zugehörige Folge der Mittelwerte.
\part Bestimmen Sie $E(\bar{X}_n)$ und $Var(\bar{X}_n)$. \textbf{(2 Punkte)}\\
\begin{solution}
$$E(\bar{X}_n) = \frac{1}{\bar{X}} ;\quad
Var(\bar{X}_n) = \frac{1}{\bar{X}^2} $$
\end{solution}
\part Wie ist $Z_n= \cos(\pi*\bar{X}_n)$ für großes $n$ approximativ verteilt? \textbf{(5 Punkte)}\\
\begin{solution}
der $\cos$ verteilt alle Ereignisse auf $[-1,1]$ und damit approximativ auf 0 für große n
\end{solution}
\end{parts}
\question Aufgabe 5: Uniforme Verteilung
Gegeben sei eine Zufallsvariable $X\sim Unif(-0.5,0.5)$.
\begin{parts}
\part Geben Sie die Wahrscheinlichkeitsdichte und die Verteilungsfunktion von $X$ an. \textbf{(2 Punkte)}
\begin{solution}
Wahrscheinlichkeitsdichte $$f(x)=\frac{1}{b-a} = \frac{1}{-0,5 - 0,5} = -1$$ \\
Verteilungsfunktion $$F(x)=\frac{x-a}{b-a} = -x+0,5$$
\end{solution}
\part Geben Sie den Median, den Erwartungswert und die Varianz von $X$ an. \textbf{(3 Punkte)}\\
\begin{solution}
Median $$Median(X)=F^{-1}(X)=\frac{a+b}{2}= 0/2 = 0$$
Erwartungswert $$E(X)=\int_{-\infty}^{\infty} xf(x) dx = \frac{a+b}{2} = 0/2 = 0$$
Varianz $$Var(X)=E(X^2)-(E(X))^2 = \frac{1}{12}(b-a)^2 = \frac{1}{12}$$
\end{solution}
\part Berechnungen im Rahmen von Bankgeschäften ergeben oft Ergebnisse mit gebrochenen Centanteilen, welche für Buchungsvorgänge gerundet werden müssen. Zum Beispiel würde man einen Betrag von $7,35...$ Cent auf 7 Cent abrunden und einen Betrag von $15.87...$ Cent auf 16 Cent aufrunden. Den Rundungsfehler kann man als uniform-verteilt auf $(-0.5,0.5)$ (eigentlich $(-0.5,0.5]$) modellieren. Nehmen Sie nun an, dass in einer Bank 106 unabhängig und identisch $Unif(-0.5,0.5)$-verteilte Rundungsvorgänge stattfinden.
\part Geben Sie mit Hilfe der Ungleichung von Tschebyscheff eine sinnvolle obere Abschätzung für die Wahrscheinlichkeit an, dass der Absolutbetrag der Summe der Rundungsfehler mindestens 10 Euro (1000 Cent) beträgt. \textbf{(2 Punkte)}
\begin{solution}
$1000 Cent / 106 \approx 9,4 Cent \rightarrow \pm 4,7 Cent$
$\mu= 0$
$\sigma= \sqrt{\frac{1}{12}} = 0,289$
$k = 4,7 Cent / \sigma = \frac{4,7}{0,289} = 16,26$
$P(|X-\mu|)\geq k*\sigma) \leq \frac{1}{k^2} \Rightarrow P(|X|\geq 16,26*0,289)\leq \frac{1}{16,26^2} = P(|X|\geq 4,7) \leq 0.061$
\dots ?
\end{solution}
\part Mit welcher Wahrscheinlichkeit verliert die Bank mindestens einen Euro (100 Cent)? Nutzen Sie den zentralen Grenzwertsatz, um diese Wahrscheinlichkeit geeignet zu approximieren. Runden Sie Ihr Ergebnis mit Hilfe der unten angegebenen Tabelle auf volle 10\%. \textbf{(3 Punkte)}\\
\begin{solution}
Grenzwertsatz $$lim_{n\rightarrow \infty} P(\sqrt{n} * \frac{\bar{X}_n -\mu}{\sigma}\leq x)=\phi (x)$$
Quantile der Standardnormalverteilung
\begin{center}
\begin{tabular}{c | c | c | c | c | c | c | c | c | c}
$\alpha$ & 10\% & 20\% & 30\% & 40\% & 50\% & 60\% & 70\% & 80\% & 90\% \\\hline
$q_{\alpha}$ & $-1,28$ & $-0,84$ & $-0,52$ & $-0,25$ & $0$ & $0,25$ & $0,52$ & $0,84$ & $1,28$
\end{tabular}
\end{center}
\end{solution}
\end{parts}
\question Aufgabe 6: Normalverteilung
Gegeben seien die unabhängigen Zufallsvariablen $X_1,X_2,...,X_n$ mit den Verteilungen $X_i \sim \mathcal{N}(\mu,9)$ für $i= 1,2,...,n$ und einem reellen Parameter $\mu$.
\begin{parts}
\part Geben Sie einen erwartungstreuen Schätzer $\hat{\mu}$ für $\mu$ an. \textbf{(1 Punkt)}\\
\begin{solution}
\end{solution}
\part Wie ist $\hat{\mu}$ verteilt? \textbf{(2 Punkte)}\\
\begin{solution}
\end{solution}
\part Geben Sie mit Hilfe von $\hat{\mu}$ und der Faustformel ein 95\%-Konfidenzintervall für $\mu$ an. \textbf{(2 Punkte)}\\
\begin{solution}
\end{solution}
\part Wie würden Sie die Hypothese $\mu= 1$ zum Niveau 5\% testen? \textbf{(2 Punkte)}\\
\begin{solution}
\end{solution}
\part Berechnen Sie $Cov(X_1+X_2,X_2+X_3)$. \textbf{(2 Punkte)}\\
\begin{solution}
\end{solution}
\part Sind $X_1+X_2$ und $X_2+X_3$ stochastisch unabhängig? \textbf{(1 Punkt)}\\
\begin{solution}
\end{solution}
\end{parts}
\question Aufgabe 7: Sinus-Verteilung
\begin{parts}
\part Wann ist eine Funktion $f:R\rightarrow R$ eine Wahrscheinlichkeitsdichte? \textbf{(2 Punkte)}\\
\begin{solution}
\end{solution}
Gegeben sei $f:R\rightarrow R$ mit $f(x) =a*\sin(x)*1_{(0,\pi)}(x)$, wobei a ein reeller Parameter ist.
\part Bestimmen Sie a so, dass f eine Wahrscheinlichkeitsdichte ist. \textbf{(1 Punkt)}\\
\begin{solution}
\end{solution}
Sei nun X eine Zufallsvariable, die die Wahrscheinlichkeitsdichte f besitzt.
\part Berechnen Sie $P(X >\frac{\pi}{2})$. \textbf{(2 Punkte)}\\
\begin{solution}
\end{solution}
\part Berechnen Sie $E X$. \textbf{(2 Punkte)}\\
\begin{solution}
\end{solution}
\part Begründen Sie, dass hier die Markoff-Ungleichung angewendet werden kann und verwenden Sie sie, um $P(X >\frac{\pi}{2})$ abzuschätzen. Wie erklären Sie den Unterschied zu c)? \textbf{(3 Punkte)}\\
\begin{solution}
\end{solution}
\end{parts}
\question Aufgabe 8: Deskriptive Statistik
Aus einer Charge von Fäden werden 5 Stück entnommen, um sie auf Reißfestigkeit zu testen. Notiert werden die erreichten Dehnungslängen $L_i,i= 1,...,5$ in cm zum Zeitpunkt des Reißens. Die Ergebnisse lauten:
\begin{center}
\begin{tabular}{c | c | c | c | c}
$L_1$ & $L_2$ & $L_3$ & $L_4$ & $L_5$ \\\hline
4 & 11 & 1 & 6 & 3
\end{tabular}
\end{center}
\begin{parts}
\part Befinden sich diese Daten auf einer Verhältnisskala? \textbf{(1 Punkt)}\\
\begin{solution}
Die Daten befinden sich auf der Ordinal-Skala, da diese benannt und in einer natürlichen Ordnung existieren.
\end{solution}
\part Skizzieren Sie die empirische Verteilungsfunktion zu diesem Datensatz. \textbf{(1 Punkt)}\\
\begin{solution}
$$F(x_i)=\sum_{j=1}^i \frac{n_j}{n}$$
Bsp für 6: $$F(6) = \sum_{j=1}^6 \frac{n_j}{n} = \frac{1}{5}+ \frac{3}{5}+ \frac{4}{5}+ \frac{6}{5} = 2,8$$
\vspace{.5cm}
\begin{tikzpicture}[
dot/.style = {
draw,
fill = white,
circle,
inner sep = 0pt,
minimum size = 4pt
}
]
\draw[thick,->] (0,0) -- (13,0) node[anchor=south west] {X};
\draw[thick,->] (0,0) -- (0,6) node[anchor=south west] {Y};
\foreach \x in {0,1,2,3,4,5,6,7,8,9,10,11,12}
\draw (\x cm,1pt) -- (\x cm,-1pt) node[anchor=north] {$\x$};
\foreach \y in {0,1,2,3,4,5}
\draw (1pt,\y cm) -- (-1pt,\y cm) node[anchor=east] {$\y$};
\draw[gray, thick] (1,1) -- (3,1);
\draw[gray, thick] (3,2) -- (4,2);
\draw[gray, thick] (4,3) -- (6,3);
\draw[gray, thick] (6,4) -- (11,4);
\draw[gray, thick] (11,5) -- (12,5);
\node at (1,1) [circle,fill=black] {};
\node at (3,2) [circle,fill=black] {};
\node at (4,3) [circle,fill=black] {};
\node at (6,4) [circle,fill=black] {};
\node at (11,5) [circle,fill=black] {};
\end{tikzpicture}
\end{solution}
\part Bestimmen Sie den Mittelwert, den Median, die Quartile und den Interquartilsabstand der Daten. Wie erklärt sich der Unterschied zwischen Median und Mittelwert? \textbf{(4 Punkte)}\\
\begin{solution}
Mittelwert: $$x_{mit}=\frac{1}{n} \sum_{i=1}^n x_i = 5$$
Median: $x_{med} \geq 50\% \text{ aller Werte } \Rightarrow x_{med}= 4$
Quartile: $$Q_{0,75} = 1,75*x_{mit} = 8,75;\quad Q_{0,25} = 0,25*x_{mit}= 1,25$$
Interquartilsabstand: $IQS=Q_{0,75} - Q_{0,25} = 8,75 - 1,25 = 7,5$
\end{solution}
\part Skizzieren Sie einen Boxplot zu diesem Datensatz. \textbf{(3 Punkte)}\\
\begin{solution}
\vspace{.5cm}
\begin{tikzpicture}
\begin{axis}
[
ytick={1,2,3},
yticklabels={Index 0, Index 1, Index 2},
]
\addplot+[
boxplot prepared={
median=4,
upper quartile=8.75,
lower quartile=1.25,
upper whisker=11,
lower whisker=1
},
] coordinates {};
\end{axis}
\end{tikzpicture}
\end{solution}
\end{parts}
\end{questions}
\end{document}